1
$\begingroup$

In a question in StackExchange (https://math.stackexchange.com/questions/4236635/sum-of-quotients-of-gamma-functions), I asked if there is a closed expression for the following sum related with a paper I am working in: $$\sum_{n=1}^N \frac{\Gamma(Ln)}{\Gamma(Ln+r)},$$ where $L,N$ are positive integers greater than 1 and $r$ is a non-integer with $1<r<2$.

The natural conversion of this sum into an integral, as pointed out in my question, doesn't seem to help at all, since I can't compute it.

I was told that the sum is related to the Fox–Wright function. That is useful in the sense that at least I can give a name to my expression, but of course doesn't help to compute the sum, so I tried to write it as $$\sum_{n=1}^N \frac{1}{(Ln)_{r}},$$ which is exactly what appears in http://specialfunctionswiki.org/index.php/Sum_of_reciprocal_Pochhammer_symbols_of_a_fixed_exponent with $L=1$. Does somebody know about a generalization of this result?

Anyway, it would suffice for me to compute the infinite sum, that is $$\sum_{n=1}^\infty \frac{\Gamma(Ln)}{\Gamma(Ln+r)},$$ which numerically I see that converges, and to know just the asymptotic expansion of a very similar expression, $$\sum_{n=1}^N \frac{\Gamma(Ln+1)}{\Gamma(Ln+r)}$$ as $N\to\infty$. This last sum doesn't converge, but I know numerically that its limit when we add some other functions of $N$ exists, so having an asymptotic expansion would be enough.

Thank you so much.

$\endgroup$
2
  • 1
    $\begingroup$ There are at least two methods to try, neither of which I have the inclination to try. (1) Euler-Maclaurin and (2) Stirling asymptotics for the summand. Since the summand is not oscillating, I'd be surprised if this is a difficult problem (at least for fixed L). $\endgroup$
    – skbmoore
    Sep 3, 2021 at 16:28
  • $\begingroup$ Dear @skbmoore, thanks for your answer. I'm not sure what you mean by applying asymptotics for the summand. I could understand it if, for instance, we were interested in what happens when $L\to\infty$, but here we consider $N\to \infty$. I think I'm missing some point. Do you have any reference on Stirling asymptotics for obtaining sums? Thank you again! $\endgroup$ Sep 3, 2021 at 17:53

1 Answer 1

1
$\begingroup$

The identity you link on the special functions wiki can be rewritten as $$\sum_{k=1}^n \frac{\Gamma(k)}{\Gamma(k+r)} = \frac{1}{(r-1)\Gamma(r)} - \frac{n\Gamma(n)}{(r-1)\Gamma(n+r)}$$

This clearly has the form of a telescoping sum: i.e. $$\sum_{k=1}^n (T(k) - T(k-1)) = T(n) - T(0)$$

For $L=1$, then, we have $T(n) = \frac{-\Gamma(n+1)}{(r-1)\Gamma(n+r)}$. With the aid of Wolfram Alpha, we get

$L=2$: $$T(n) = -\frac{\Gamma(2n+2)}{\Gamma(2n+r+2)} {}_3F_2\left(\begin{matrix} 1, n+1, n+\tfrac32 \\ n + \tfrac r2 + 1, n + \tfrac r2 + \tfrac32 \end{matrix} \middle\vert 1\right)$$

$L=3$: $$T(n) = - \frac{\Gamma(3n+3)}{\Gamma(3n+r+3)} {}_4F_3\left(\begin{matrix} 1, n + 1, n + \tfrac43, n + \tfrac53 \\ n + \tfrac r3 + 1, n + \tfrac r3 + \tfrac43, n + \tfrac r3 + \tfrac 53 \end{matrix} \middle\vert 1\right)$$

and in general $$T(n) = - \frac{\Gamma(Ln+L)}{\Gamma(Ln+r+L)} {}_{L+1}F_L\left(\begin{matrix} 1, n + 1, n + \tfrac{L+1}L, \ldots, n + \tfrac{2L-1}L \\ n + \tfrac rL + 1, n + \tfrac rL + \tfrac{L+1}L, \ldots, n + \tfrac rL + \tfrac{2L-1}L \end{matrix} \middle\vert 1\right)$$

So the answer is essentially negative: that sum doesn't seem to generalise in a useful way, because for $L > 1$ the hypergeometric form is really just a reformulation of the Fox-Wright form.

$\endgroup$
1
  • $\begingroup$ Thank you very much, Peter, good point! I had bad feelings about it but wasn't completely sure. $\endgroup$ Sep 3, 2021 at 18:01

Your Answer

By clicking “Post Your Answer”, you agree to our terms of service and acknowledge you have read our privacy policy.

Not the answer you're looking for? Browse other questions tagged or ask your own question.